5
$\开始组$

根据维基百科,可以颠倒Binet的斐波那契数公式:

$$F_n=\frac{\varphi^n-\psi^n}{\varpi-\psi}=\frac{\varfi^n-\psi ^n}{\sqrt 5}$$哪里$\varphi=\frac{1+\sqrt{5}}{2}\约1.61803\,39887\ldot$$\psi=\frac{1-\sqrt{5}}{2}=1-\varphi=-{1\over\varphi}\近似-0.61803\,39887\ldots$,或者更具体地说是其截断变体:

$$F_n=\left\lfloor\frac{\varphi^n}{\sqrt 5}+\frac{1}{2}\right\rfloor$$

查找索引n美元(法郎)$不大于实数的最大斐波那契数$F>1$:

$$n(F)=\left\lfloor\log_\varphi(F\sqrt5+1/2)\right\rfloor$$

由于没有给出理由,除了楼层函数是单调的之外,我想证明这一点。

因此,假设截断公式成立:$$F_n=\left\lfloor\frac{\varphi^n}{\sqrt 5}+\frac{1}{2}\right\rfloor$$ $$F_n=\frac{\varphi^n}{\sqrt 5}+\frac{1}{2}+E,\text{其中$0\le E<1$}$$ $$\varphi^n=\sqrt5(F_n-\frac{1}{2}-E),\text{其中$0\le E<1$}$$ $$n\log\varphi=\log(\sqrt5(F_n-\frac{1}{2}-E)),\text{其中$0\le E<1$}$$ $$n=\log_\varphi(\sqrt5(F_n-\frac{1}{2}-E)),\text{其中$0\le E<1$}$$

由于n是一个整数,我们可以发言:

$$n=\lfloor n\rfloor=\left\lfloor\log_\varphi(\sqrt5(F_n-\frac{1}{2}-E))\right\rfloor,\text{其中$0\le E<1$}$$ $$n=\left\lfloor\log_\varphi(\sqrt5 F_n\left(1-\frac{1+2E}{2F_n}\right)\right\rfloor,\text{其中$0\le E<1$}$$ $$n=\left\lfloor\log_\varphi(\sqrt5 F_n)+\log_\ varphi\left(1-\frac{1+2E}{2F_n}\right)\right\rfloor,\text{其中$0\le E<1$}$$

这似乎不是一条正确的道路。。。

$\端组$

1答案1

重置为默认值
6
$\开始组$

我们有$$F_n=\frac{\phi^n-(-\frac{1}{\phi})^n}{\sqrt{5}}$$什么可以反转书写$$\斐^{2n}-F_n\sqrt{5}\,\phi^n-(-1)^n=0$$它是平方英寸美元\phi^n$.所以$$\phi^n=\frac{F_n\sqrt{5}+\sqrt}5F_n^2\pm4}}{2}\暗示n_pm\log(\phi)=\log\Bigg[\frac}F_n\scrt{5}+\sqrt[2\pm4}{2{2}\Bigg]$$两种解决方案之一是整数$$\剩余(\开始{数组}{cccc}n&F_n&n_-&n_+\\3 & 2 & 3.000000000 & 3.209573980 \\4 & 3 & 3.907487979 & 4.000000000 \\5 & 5 & 5.000000000 & 5.033256487 \\6 & 8 & 5.987011534 & 6.000000000 \\7 & 13 & 7.000000000 & 7.004918572 \\8 & 21 & 7.998115113 & 8.000000000 \\9 & 34 & 9.000000000 & 9.000719061 \\10 & 55 & 9.999725212 & 10.00000000\结束{数组}\右侧)$$

如果不是这样,那么这个数字就不是斐波那契。正在尝试$12345678987654321$,结果是$n_pm=78.669724$; 所以,它不是斐波那契数(但我们知道最接近的一个)。$$F_{78}=8944394323791464\quad<\quad 1234567898765431$$ $$F_{79}=14472334024676221\quad>\quad 12345678987654321$$

编辑

考虑两个根之间的区别:考虑$F_n$做大并进行系列扩展$$n_+-n_-\sim\frac{2}{5F_n^2\log(\phi)}<\frac{1}{F_n^2}$$

$\端组$
2
  • $\开始组$ 为什么我们可以忽略abc公式给出的另一个根? $\端组$
    – 香港Bst
    2021年12月24日15:11
  • $\开始组$ 如果忽略+-4,那么可以将RHS简化为log(F*sqrt5)。取整将恢复n-或n+(以整数为准)。四舍五入与加上1/2并占用地板相同。这应该能让你理解问题中的公式,但我不相信它具有此处描述的属性。。。 $\端组$
    – 香港Bst
    2022年1月3日11:52

你必须登录来回答这个问题。

不是你想要的答案吗?浏览标记的其他问题.